Difference between revisions of "2013 USAMO Problems/Problem 6"
m (→Solution) |
|||
(One intermediate revision by one other user not shown) | |||
Line 26: | Line 26: | ||
<cmath>x^2 - ax + \frac{a^2 bc}{(b+c)^2} = 0,</cmath> | <cmath>x^2 - ax + \frac{a^2 bc}{(b+c)^2} = 0,</cmath> | ||
which factors as | which factors as | ||
− | <cmath>(x - \frac{ab}{b+c})(x - \frac{ac}{b+c}) = 0.</cmath> | + | <cmath>\left(x - \frac{ab}{b+c}\right)\left(x - \frac{ac}{b+c}\right) = 0.</cmath> |
Hence, <math>PB = \frac{ab}{b+c}</math> or <math>\frac{ac}{b+c}</math>, and so the <math>P</math> corresponding to these lengths are our answer. | Hence, <math>PB = \frac{ab}{b+c}</math> or <math>\frac{ac}{b+c}</math>, and so the <math>P</math> corresponding to these lengths are our answer. | ||
+ | |||
{{MAA Notice}} | {{MAA Notice}} |
Latest revision as of 17:29, 10 May 2023
Problem
Let be a triangle. Find all points on segment satisfying the following property: If and are the intersections of line with the common external tangent lines of the circumcircles of triangles and , then
Solution
Let circle (i.e. the circumcircle of ), be with radii , and centers , respectively, and be the distance between their centers.
Lemma.
Proof. Let the external tangent containing meet at and at , and let the external tangent containing meet at and at . Then clearly and are parallel (for they are both perpendicular ), and so is a trapezoid.
Now, by Power of a Point, and so is the midpoint of . Similarly, is the midpoint of . Hence, Let , meet s at , respectively. Then by similar triangles and the Pythagorean Theorem we deduce that and . But it is clear that , is the midpoint of , , respectively, so as desired.
Lemma 2. Triangles and are similar.
Proof. and similarly , so the triangles are similar by AA Similarity.
Also, let intersect at . Then obviously is the midpoint of and is an altitude of triangle .Thus, we can simplify our expression of : where is the length of the altitude from in triangle . Hence, substituting into our condition and using gives Using by Heron's Formula (where is the area of triangle , our condition becomes which by becomes Let ; then . The quadratic in is which factors as Hence, or , and so the corresponding to these lengths are our answer.
The problems on this page are copyrighted by the Mathematical Association of America's American Mathematics Competitions.